Verbal questions from any Manhattan Prep GMAT Computer Adaptive Test. Topic subject should be the first few words of your question.
akhp77
Students
 
Posts: 114
Joined: Wed Mar 24, 2010 7:25 pm
 

Profits for one of Company

by akhp77 Tue May 18, 2010 2:16 am

Source: MGMAT CAT

Profits for one of Company X's flagship products have been declining slowly for several years. The CFO investigated and determined that inflation has raised the cost of producing the product but consumers who were surveyed reported that they weren't willing to pay more than the current price. As a result, the CFO recommended that the company stop producing this product because the CEO only wants products whose profit margins are increasing.

The answer to which of the following questions would be most useful in evaluating whether the CFO's decision to divest the company of its flagship product is warranted?

A: Does the company have new and profitable products available with which to replace the flagship product?

B: Will the rest of Company X's management team agree with the CFO's recommendation?

C: Are there additional features which could be added to the product and for which consumers might be willing to pay a higher price?

D: Is there a way to alter the manufacturing or distribution processes in order to reduce the cost to produce the flagship product?

E: What percentage of Company X's revenues is represented by sales of the flagship product in question?

OA: D

Initially, I selected A but when I saw answer D make sense.
What would be the criteria for A to reject?
Is A a general statement and D focuses in cost of manufacturing process?
tim
Course Students
 
Posts: 5665
Joined: Tue Sep 11, 2007 9:08 am
Location: Southwest Airlines, seat 21C
 

Re: Profits for one of Company

by tim Tue Jun 08, 2010 5:28 pm

The best thing to do with this one is to recognize that A is totally irrelevant. We are concerned about declining profits of one product, so knowing whether there are other products to sell will have nothing to do with whether we keep selling the one product unless we are told that there is a limited production capacity to begin with. D is more directly relevant to the issue of profitability of the particular product in question..
Tim Sanders
Manhattan GMAT Instructor

Follow this link for some important tips to get the most out of your forum experience:
https://www.manhattanprep.com/gmat/forums/a-few-tips-t31405.html
akhp77
Students
 
Posts: 114
Joined: Wed Mar 24, 2010 7:25 pm
 

Re: Profits for one of Company

by akhp77 Wed Jun 09, 2010 2:37 am

Thanks Tim
mschwrtz
ManhattanGMAT Staff
 
Posts: 498
Joined: Tue Dec 14, 2004 1:03 pm
 

Re: Profits for one of Company

by mschwrtz Tue Jun 29, 2010 2:53 am

You're welcome.
s.ashwin.rao
Students
 
Posts: 40
Joined: Sat Nov 27, 2010 4:38 pm
 

Re: Profits for one of Company

by s.ashwin.rao Mon Jan 03, 2011 7:29 am

But tim,
The CFO has already decided to stop production of this particular product so why then should we even bothered if we can reduce the cost, I would rather go A which could help me look for alternate products.

Thanks
gaurav.pdhyy
Students
 
Posts: 10
Joined: Sun Aug 01, 2010 9:55 pm
 

Re: Profits for one of Company

by gaurav.pdhyy Mon Jan 03, 2011 10:10 am

Hi Ron/Tim/Stacey , Can someone pls explain why C is ignored. Here the CFO asks to divest the flagship of a product, to which consumers are not willing to pay a higher price. But there's hasn't been any mention that the consumer's are totally not planning to pay a price if its been modified for features which they anticipate to have. Under current product they might be reluctant to pay such a price. So C is also a heavy duty contender. Pls provide inputs.

Thanks
dmitryknowsbest
ManhattanGMAT Staff
 
Posts: 78
Joined: Wed Jul 29, 2009 8:50 am
 

Re: Profits for one of Company

by dmitryknowsbest Tue Jan 04, 2011 12:52 am

s.ashwin.rao,

We have been asked to evaluate the CFO's decision. The product has not yet been discontinued. These "evaluate the argument" questions are basically neutral versions of strengthen/weaken. We are looking for a question that, if answered, would either strengthen or weaken the argument (it doesn't matter which, as long as it helps us to evaluate the validity of the argument). Let's look at what happens if we get answers to A and D.

A==> If we get an answer of YES, that lets us know that there are alternatives available. That's good news for the company IF the flagship product is discontinued, but it does nothing to tell us if discontinuing that product is a good idea.

If we get an answer of NO, our response might be "Okay, then it's a bad idea to discontinue the current product." But remember the other premise involved--the CEO only wants products with increasing profit margins, and our job in this problem is not to assess the CEO's reasoning. To evaluate the CFO's decision, all we need to do is determine whether there is any way for the profit margin on the flagship product to increase.

D==> This is more helpful. If we get an answer of YES, then we have a mechanism for the profit margin to increase. This would weaken the CFO's argument. If we get an answer of NO, and consumers are not willing to pay more, it looks like the product is doomed to a smaller profit margin. This would strengthen the CFO's argument. Either way, we would have information that would allow us to determine the strength of the argument.
Dmitry Farber
Manhattan GMAT Instructor
dmitryknowsbest
ManhattanGMAT Staff
 
Posts: 78
Joined: Wed Jul 29, 2009 8:50 am
 

Re: Profits for one of Company

by dmitryknowsbest Tue Jan 04, 2011 12:54 am

Gaurav,

C would allow us to determine if the revenue for the product might go up, but it doesn't tell us to what extent the cost will change. Without that information, it is hard to tell what will happen to the profit margin. In D, on the other hand, we have the possibility of the revenue staying the same while the cost goes down. If this happened, the profit margin would clearly increase.
Dmitry Farber
Manhattan GMAT Instructor
gaurav.pdhyy
Students
 
Posts: 10
Joined: Sun Aug 01, 2010 9:55 pm
 

Re: Profits for one of Company

by gaurav.pdhyy Tue Jan 04, 2011 3:43 am

Dmitry WOW for you....perfectly makes sense :) Thanks a ton
girish_ns
Course Students
 
Posts: 3
Joined: Tue Apr 20, 2010 6:05 am
 

Re: Profits for one of Company

by girish_ns Tue Jan 04, 2011 8:54 pm

Hi Dimitry,

Answer choice (D) says that the cost of the product can or cannot be lowered, but it does not tell us by how much. Does this matter? For example, the cost of production may go down by a little bit, but that may not be enough to ensure profit margins are "increasing". The profit margin will increase from the current margin though, but the profit margin may still be lower than what it was before the costs began to rise, or a few years ago.
jnelson0612
ManhattanGMAT Staff
 
Posts: 2664
Joined: Fri Feb 05, 2010 10:57 am
 

Re: Profits for one of Company

by jnelson0612 Wed Jan 05, 2011 9:45 am

girish_ns Wrote:Hi Dimitry,

Answer choice (D) says that the cost of the product can or cannot be lowered, but it does not tell us by how much. Does this matter? For example, the cost of production may go down by a little bit, but that may not be enough to ensure profit margins are "increasing". The profit margin will increase from the current margin though, but the profit margin may still be lower than what it was before the costs began to rise, or a few years ago.


To answer your question, no, this does not matter. We don't need to get into the details here.

I think you are analyzing this a bit too much. D is simply going to tell us whether the product can be brought to market in a less expensive manner. If it can, then maybe it will meet the CEO's requirement of a product with increasing profit margins. That is why it would be helpful to know whether this is feasible before the decision is made to discontinue this product.
Jamie Nelson
ManhattanGMAT Instructor
aagar2003
Students
 
Posts: 24
Joined: Sat Mar 06, 2010 10:20 am
 

Re: Profits for one of Company

by aagar2003 Thu Jun 09, 2011 8:32 am

dmitryknowsbest Wrote:s.ashwin.rao,

D==> This is more helpful. If we get an answer of YES, then we have a mechanism for the profit margin to increase. This would weaken the CFO's argument. If we get an answer of NO, and consumers are not willing to pay more, it looks like the product is doomed to a smaller profit margin. This would strengthen the CFO's argument. Either way, we would have information that would allow us to determine the strength of the argument.


I have a problem with your reasoning. Question clearly mentions which answer would be most useful in evaluating CFO's decision to divest .... is warranted? The question is tough to understand what is 'warranted'.
The correct answer should be the choice that mentions there are no ways of reducing the cost any further and also there is an alternative product that is new and profitable that will be the new flagship product of the company.

Your explanation breaks down in a scenario in which no improvements could be made to reduce the cost and there aren't any better products too to subsitute the original flagship product. In such a scenario, the CFO should better make the management team aware of such a scenario that there is a dead end for the company and we possibly need to liquidate.

Now the answer choices:
A: Does the company have new and profitable products available with which to replace the flagship product?
CFO has already come up with a logic on when to divest. The question is about evaluating whether CFO's logic of divesting is warranted or not. Note: He has not made a decision yet. The question asks about no matter what happens, will his alternative strategy work. If yes, then how do we MAKE SURE before he makes a decision. This should be CORRECT answer because irrespective of whatever happens with the existing product, his decision is warranted if there exists other product which is profitable (however in general business sense, a CFO would make decison of long-term cash flow and not a point of time profit margin as decsribed in the problem) CORRECT.
B: Will the rest of Company X's management team agree with the CFO's recommendation?
This is also an interesting answer choice as explained above. If the management does not approve of divesting into a new flagship product, CFO's decision cannot be warranted. BUT it is nowhere specified in the question who the final authority is and whether the management team also need to approve it or not. AGAIN in reality, while moving away from an established product would require it but as per question for GMAT, I will limit my business sense. INCORRECT.
C: Are there additional features which could be added to the product and for which consumers might be willing to pay a higher price?
This is one of the ways of checking whether existing flagship product can be made profitable but does not warranty whether CFO's logic. Also higher price of the product that consumer is willing to pay does not mean that will be higher than the offset due to inflation. INCORRECT.
D: Is there a way to alter the manufacturing or distribution processes in order to reduce the cost to produce the flagship product?
This is one of the ways of checking whether existing flagship product can be made profitable but does not warranty whether CFO's logic.
Also, are these the only costs that are left for CFO to optimize. Since there is an OR, can't it be possible that he decreases manufacturing cost by moving to Arkansas but distribution costs increases because of moving to remote location. The correct alternative should have whether he is able to reduce the costs any further or not. INCORRECT.

E: What percentage of Company X's revenues is represented by sales of the flagship product in question?
Here we want to talk about costs and profit margin. Revenue is a function of number of pieces sold and selling price. This does not help. INCORRECT.


Correct me where am I wrong?
messi10
Course Students
 
Posts: 320
Joined: Thu Dec 02, 2010 2:18 am
 

Re: Profits for one of Company

by messi10 Thu Jun 09, 2011 11:37 am

Hi Ashish,

You seem more interested in solving the problem for the CFO than answering the question. Please bear in mind that with CR, you are given an argument which is 3-4 lines long. You may not like whats in those arguments, but you have to adhere to the information provided. You are allowed to make logical assumptions when getting to the answer but not drastic ones that sound better. Your reasoning is making far too many assumptions and you are making them based on your opinions or experiences.

I will try and point out some issues with your explanation of Choice A:

CFO has already come up with a logic on when to divest.

He has made a recommendation but where does it say "when"? Yes, there may be a "when" in that recommendation but is that relevant to answer this question?


The question is about evaluating whether CFO's logic of divesting is warranted or not.

Ok...I would use the word decision instead of logic

Note: He has not made a decision yet.

He has put forward a recommendation and the question clearly asks whether his decision is justified. I can see how you got to this: He has put forward a recommendation but in his mind, he may not have decided. Possible and arguable but once again, is this really relevant?


The question asks about no matter what happens, will his alternative strategy work.

This is where you start going off track. There is no mention of an alternative strategy in the argument.


If yes, then how do we MAKE SURE before he makes a decision.

And now you that you have already gone off track, this is going even further.


This should be CORRECT answer because irrespective of whatever happens with the existing product, his decision is warranted if there exists other product which is profitable

Even though you are wrong about this being the correct answer, this is the only line that makes some sense based on what Choice A says.


(however in general business sense, a CFO would make decison of long-term cash flow and not a point of time profit margin as decsribed in the problem) CORRECT.

Once again, no need to think this far. But I think you may have just put this out as a thought... fair enough


Stick to the argument and what the question asks. Do not make far fetched assumptions. You will suffer on the GMAT if you do. This post has been answered by almost 4 instructors. Its good to question reasons behind wrong answers and pick their brains on the whys. But refuting all of them together in the same post is just a waste of time. Not saying that there is anything wrong with asking questions, but questioning the right answer on the GMAT with such conviction is again a waste of time. You will see many posts where instructors have clearly said: The right answer on the GMAT is always right. It that were not true, GMAT would be facing law suits for the rest of its foreseeable existence.

My advice to you is to go through the CR guide, if you have not already done so and try and get the right mindset to answer these type of questions. You must attain the discipline to not bring in external knowledge to the point that it starts harming you.

Regards

Sunil
jnelson0612
ManhattanGMAT Staff
 
Posts: 2664
Joined: Fri Feb 05, 2010 10:57 am
 

Re: Profits for one of Company

by jnelson0612 Sun Jun 12, 2011 11:23 pm

Very, very wise words Sunil. Ashish, I hope you are more squared away now.
Jamie Nelson
ManhattanGMAT Instructor
Harisankar108
Students
 
Posts: 1
Joined: Sat Mar 28, 2015 12:35 am
 

Re: Profits for one of Company

by Harisankar108 Mon May 04, 2015 2:05 am

Hi,

I came across this question in my mock exam.

Please explain what is wrong with option E. The explanation for option E states that if the product is a large contributor of revenue, then there is trouble in dropping the product.

This makes me think which part to concentrate - the portion suggesting discontinue the product or the portion suggesting discontinue products not achieving increasing margins.
The explanation seems to tell me that the latter is the focus, but how do I arrive at this?

Thanks,
Hari